Những câu hỏi liên quan
Rose Princess
Xem chi tiết
Thắng Nguyên
Xem chi tiết
Kiệt Nguyễn
28 tháng 8 2020 lúc 9:36

Áp dụng giả thiết và một đánh giá quen thuộc, ta được: \(16\left(a+b+c\right)\ge\frac{1}{a}+\frac{1}{b}+\frac{1}{c}=\frac{ab+bc+ca}{abc}=\frac{\left(ab+bc+ca\right)^2}{abc\left(ab+bc+ca\right)}\ge\frac{3\left(a+b+c\right)}{ab+bc+ca}\)hay \(\frac{1}{6\left(ab+bc+ca\right)}\le\frac{8}{9}\)

Đến đây, ta cần chứng minh \(\frac{1}{\left(a+b+\sqrt{2\left(a+c\right)}\right)^3}+\frac{1}{\left(b+c+\sqrt{2\left(b+a\right)}\right)^3}+\frac{1}{\left(c+a+\sqrt{2\left(c+b\right)}\right)^3}\le\frac{1}{6\left(ab+bc+ca\right)}\)

 Áp dụng bất đẳng thức Cauchy cho ba số dương ta có \(a+b+\sqrt{2\left(a+c\right)}=a+b+\sqrt{\frac{a+c}{2}}+\sqrt{\frac{a+c}{2}}\ge3\sqrt[3]{\frac{\left(a+b\right)\left(a+c\right)}{2}}\)hay \(\left(a+b+\sqrt{2\left(a+c\right)}\right)^3\ge\frac{27\left(a+b\right)\left(a+c\right)}{2}\Leftrightarrow\frac{1}{\left(a+b+2\sqrt{a+c}\right)^3}\le\frac{2}{27\left(a+b\right)\left(a+c\right)}\)

Hoàn toàn tương tự ta có \(\frac{1}{\left(b+c+2\sqrt{b+a}\right)^3}\le\frac{2}{27\left(b+c\right)\left(b+a\right)}\)\(\frac{1}{\left(c+a+2\sqrt{c+b}\right)^3}\le\frac{2}{27\left(c+a\right)\left(c+b\right)}\)

Cộng theo vế các bất đẳng thức trên ta được \(\frac{1}{\left(a+b+\sqrt{2\left(a+c\right)}\right)^3}+\frac{1}{\left(b+c+\sqrt{2\left(b+a\right)}\right)^3}+\frac{1}{\left(c+a+\sqrt{2\left(c+b\right)}\right)^3}\le\frac{4\left(a+b+c\right)}{27\left(a+b\right)\left(b+c\right)\left(c+a\right)}\)Phép chứng minh sẽ hoàn tất nếu ta chỉ ra được \(\frac{4\left(a+b+c\right)}{27\left(a+b\right)\left(b+c\right)\left(c+a\right)}\le\frac{1}{6\left(ab+bc+ca\right)}\)\(\Leftrightarrow\left(a+b\right)\left(b+c\right)\left(c+a\right)\ge\frac{8}{9}\left(ab+bc+ca\right)\left(a+b+c\right)\)

Đây là một đánh giá đúng, thật vậy: đặt a + b + c = p; ab + bc + ca = q; abc = r thì bất đẳng thức trên trở thành \(pq-r\ge\frac{8}{9}pq\Leftrightarrow\frac{1}{9}pq\ge r\)*đúng vì \(a+b+c\ge3\sqrt[3]{abc}\)\(ab+bc+ca\ge3\sqrt[3]{\left(abc\right)^2}\))

Vậy bất đẳng thức được chứng minh

Đẳng thức xảy ra khi \(a=b=c=\frac{1}{4}\)

Bình luận (0)
 Khách vãng lai đã xóa
Lê Văn Hoàng
Xem chi tiết
Kiệt Nguyễn
23 tháng 1 2021 lúc 13:15

\(\text{⋄}\)Dễ có: \(B\ge\left(3+\frac{4}{a+b}\right)\left(3+\frac{4}{b+c}\right)\left(3+\frac{4}{c+a}\right)\)

\(\text{⋄}\)Đặt \(b+c=x;c+a=y;a+b=z\left(x,y,z>0\right)\)thì \(a=\frac{y+z-x}{2};b=\frac{z+x-y}{2};c=\frac{x+y-z}{2}\)

Giả thiết được viết lại thành: \(x+y+z\le3\)và ta cần tìm giá trị nhỏ nhất của \(\left(3+\frac{4}{x}\right)\left(3+\frac{4}{y}\right)\left(3+\frac{4}{z}\right)\)

\(\text{⋄}\)Ta có: \(\left(3+\frac{4}{x}\right)\left(3+\frac{4}{y}\right)\left(3+\frac{4}{z}\right)=27+36\left(\frac{1}{x}+\frac{1}{y}+\frac{1}{z}\right)+48\left(\frac{1}{xy}+\frac{1}{yz}+\frac{1}{zx}\right)+\frac{64}{xyz}\)\(\ge27+36.\frac{9}{x+y+z}+48.\frac{27}{\left(x+y+z\right)^2}+64.\frac{27}{\left(x+y+z\right)^3}\ge343\)

Đẳng thức xảy ra khi x = y = z = 1 hay a = b = c = 1/2

Bình luận (0)
 Khách vãng lai đã xóa
Lê Đình Quân
Xem chi tiết
Nguyễn Việt Lâm
22 tháng 2 2020 lúc 6:14

\(\frac{3}{2}\ge a+b+c\ge3\sqrt[3]{abc}\Rightarrow abc\le\frac{1}{8}\)

\(3+\frac{1}{a}+\frac{1}{b}=1+1+1+\frac{1}{2a}+\frac{1}{2a}+\frac{1}{2b}+\frac{1}{2b}\ge7\sqrt[7]{\frac{1}{16a^2b^2}}\)

\(\Rightarrow P\ge343\sqrt[7]{\frac{1}{16^3\left(abc\right)^4}}\ge343\sqrt[7]{\frac{1}{16^3\left(\frac{1}{8}\right)^4}}=343\)

Dấu "=" xảy ra khi \(a=b=c=\frac{1}{2}\)

Bình luận (0)
 Khách vãng lai đã xóa
Vương Thiên Nhi
Xem chi tiết
Võ Hồng Phúc
19 tháng 11 2019 lúc 17:19

Áp dụng BĐT AM - GM:

\(\frac{3}{2}\ge a+b+c\ge3\sqrt[3]{abc}\) \(\Rightarrow abc\le\frac{1}{8}\)

\(1+1+1+\frac{1}{2a}+\frac{1}{2a}+\frac{1}{2b}+\frac{1}{2b}\ge7\sqrt[7]{\frac{1}{16a^2b^2}}\)

\(\Leftrightarrow3+\frac{1}{a}+\frac{1}{b}\ge7\sqrt[7]{\frac{1}{16a^2b^2}}\)

Tương tự ta CM được:

\(3+\frac{1}{b}+\frac{1}{c}\ge7\sqrt[7]{\frac{1}{16b^2c^2}}\)

\(3+\frac{1}{c}+\frac{1}{a}\ge\ge7\sqrt[7]{\frac{1}{16c^2a^2}}\)

Nhân vế theo vế 3 bất đẳng thức trên:

\(S\ge343\sqrt[7]{\frac{1}{4096a^4b^4c^4}}\ge343\sqrt[7]{\frac{1}{4096.\frac{1}{8^4}}}=343\)

\(\Rightarrow Min_S=343\Leftrightarrow a=b=c=\frac{1}{2}\)

Bình luận (0)
 Khách vãng lai đã xóa
sjbjscb
19 tháng 11 2019 lúc 20:00
Bình luận (0)
 Khách vãng lai đã xóa
Nguyễn Minh Nguyệt
Xem chi tiết
Hoàng Thị Ánh Phương
10 tháng 3 2020 lúc 8:34

a )

Áp dụng BĐT Bunhiacopxki ta có :

\(\left(b^2+\left(c+a\right)^2\right)\left(1+\right)\ge\left(b+2\left(a+c\right)\right)^2\)

\(\Rightarrow\sqrt{\frac{a^2}{b^2+\left(c+a\right)^2}}\le\sqrt{5}.\frac{a}{b+2c+2a}\)

\(\Rightarrow VT\le\sqrt{5}.\left(\frac{a}{b+2c+2a}+\frac{b}{c+2a+2b}+\frac{c}{a+2b+2c}\right)\)

Cần chứng minh : \(\frac{a}{b+2c+2a}+\frac{b}{c+2a+2b}+\frac{c}{a+2b+2c}\le\frac{3}{5}\)

\(\Leftrightarrow\left(\frac{1}{2}-\frac{a}{b+2c+2a}\right)+\left(\frac{1}{2}-\frac{b}{c+2a+2b}\right)+\left(\frac{1}{2}-\frac{c}{a+2b+2c}\right)\ge\frac{9}{10}\)

\(\Leftrightarrow\frac{b+2c}{b+2c+2a}+\frac{c+2a}{c+2a+2b}+\frac{a+2b}{a+2b+2c}\ge\frac{9}{5}\)

Áp dụng BĐT Bunhiacopxki dạng phân thức ở vế trái :

\(\Rightarrow VT\ge\frac{\left(b+2c+c+2a+a+2b\right)^2}{\left(b+2c\right)^2+2a\left(b+2c\right)+\left(c+2a\right)^2+2b\left(c+2a\right)+\left(a+2b\right)^2+2c\left(a+2b\right)}\)

\(=\frac{9\left(a+b+c\right)^2}{5\left(a+b+b\right)^2}=\frac{9}{5}\left(đpcm\right)\)

Dấu " = '" xảy ra khi a=b=c

Bình luận (0)
 Khách vãng lai đã xóa
Hoàng Thị Ánh Phương
10 tháng 3 2020 lúc 8:44

b ) Ta có abc =1

Ta chứng minh :

\(\frac{1}{ab+a+1}+\frac{1}{bc+b+1}+\frac{1}{ac+c+1}=1\)

VT \(=\frac{1}{ab+a+1}+\frac{a}{abc+ab+a}+\frac{ab}{a^2bc+abc+ac}\)

\(=\frac{1}{ab+a+1}+\frac{a}{ab+a+1}+\frac{ab}{ab+a+1}=1\left(đpcm\right)\)

Ta có : \(\left(1+a\right)^2+b^2+5=\left(a^2+b^2\right)+2a+6\ge2ab+2a+6\)

\(\Rightarrow\frac{\left(1+a\right)^2+b^2+5}{ab+a+4}=\frac{2ab+2a+6}{ab+a+4}=2-\frac{2}{ab+a+4}\)

\(\frac{1}{ab+a+4}=\frac{1}{ab+a+1+3}\le\frac{1}{4}\left(\frac{1}{ab+a+1}+\frac{1}{3}\right)\) ( do \(\frac{1}{x+y}\le\frac{1}{4}\left(\frac{1}{x}+\frac{1}{y}\right)\)

\(\Rightarrow\frac{\left(1+a\right)^2+b^2+5}{ab+a+4}\ge2-\frac{1}{2}\left(\frac{1}{ab+a+1}+\frac{1}{3}\right)=\frac{11}{6}-\frac{1}{2}.\frac{1}{ab+a+1}\)

Khi đó :

\(P\ge\frac{11}{2}-\frac{1}{2}.\left(\frac{1}{ab+a+1}+\frac{1}{bc+b+1}+\frac{1}{ac+c+1}\right)=\frac{11}{2}-\frac{1}{2}.1=5\)

\(P_{Min}=5\) khi \(a=b=c=1\)

Bình luận (0)
 Khách vãng lai đã xóa
Lê Văn Hoàng
Xem chi tiết
Nguyễn Thiều Công Thành
16 tháng 9 2017 lúc 20:24

\(\Leftrightarrow M=\frac{bc}{a^2\left(b+c\right)}+\frac{ca}{b^2\left(c+â\right)}+\frac{ab}{c^2\left(a+b\right)}\)

áp dụng bđt cauchy ta có:

\(\frac{bc}{a^2\left(b+c\right)}+\frac{b+c}{4bc}\ge\frac{1}{a}\);\(\frac{ca}{b^2\left(c+a\right)}+\frac{c+a}{4ca}\ge\frac{1}{b}\);\(\frac{ab}{c^2\left(a+b\right)}+\frac{a+b}{4ab}\ge\frac{1}{c}\)

\(\Rightarrow M\ge\frac{1}{2a}+\frac{1}{2b}+\frac{1}{2c}\ge3\sqrt[3]{\frac{1}{8abc}}=\frac{3}{2}\)

Bình luận (0)
Nguyễn Anh Tuấn
Xem chi tiết
vu duc thanh
8 tháng 7 2016 lúc 22:36

bài 2 thì bạn áp dụng bdt cô si với lựa chọn điểm rơi  hoặc bdt holder  ( nó giống kiểu bunhia ngược ) . bai 1 thi ap dung cai nay \(\frac{1}{x}+\frac{1}{y}>=\frac{1}{x+y}\)  câu 1 khó hơn nhưng bạn biết lựa chọn điểm rơi với áp dụng bdt phụ kia là ok .

Bình luận (0)
Thắng Nguyễn
9 tháng 7 2016 lúc 8:37

Bài 1:Đặt VT=A

Dùng BĐT \(\left(x+y+z\right)\left(\frac{1}{x}+\frac{1}{y}+\frac{1}{z}\right)\ge9\Rightarrow\frac{1}{x+y+z}\le\frac{1}{9}\left(\frac{1}{x}+\frac{1}{y}+\frac{1}{z}\right)x,y,z>0\)

Áp dụng vào bài toán trên với x=a+c;y=b+a;z=2b ta có:

\(\frac{ab}{a+3b+2c}=\frac{ab}{\left(a+c\right)+\left(b+c\right)+2b}\le\frac{ab}{9}\left(\frac{1}{a+c}+\frac{1}{b+c}+\frac{1}{2b}\right)\)

Tương tự với 2 cái còn lại

\(A\le\frac{1}{9}\left(\frac{bc+ac}{a+b}+\frac{bc+ab}{a+c}+\frac{ab+ac}{b+c}\right)+\frac{1}{18}\left(a+b+c\right)\)

\(\Rightarrow A\le\frac{1}{9}\left(a+b+c\right)+\frac{1}{18}\left(a+b+c\right)=\frac{a+b+c}{6}\)

Đẳng thức xảy ra khi a=b=c 

Bài 2:

Biến đổi BPT \(4\left(\frac{a^3}{\left(1+b\right)\left(1+c\right)}+\frac{b^3}{\left(1+c\right)\left(1+a\right)}+\frac{c^3}{\left(1+a\right)\left(1+b\right)}\right)\ge3\)

\(\Rightarrow\frac{a^3}{\left(1+b\right)\left(1+c\right)}+\frac{b^3}{\left(1+c\right)\left(1+a\right)}+\frac{c^3}{\left(1+a\right)\left(1+b\right)}\ge\frac{3}{4}\)

Dự đoán điểm rơi xảy ra khi a=b=c=1

\(\frac{a^3}{\left(1+b\right)\left(1+c\right)}+\frac{1+b}{8}+\frac{1+c}{8}\ge\frac{3a}{4}\)

Tương tự suy ra

\(VT\ge\frac{2\left(a+b+c\right)-3}{4}\ge\frac{2\cdot3\sqrt{abc}-3}{4}=\frac{3}{4}\)

Bình luận (0)
o0o I am a studious pers...
Xem chi tiết
Lầy Văn Lội
9 tháng 5 2017 lúc 0:24

để biểu thức cho đơn giản , ta đặt x=a+1,y=b+1,z=c+1(x,y,z>0)

thì giả thiết thành \(\frac{1}{x+1}+\frac{3}{y+3}\le\frac{z}{z+2}\) .Tìm min xyz 

Áp dụng bất đẳng thức cauchy:\(\frac{z}{z+2}\ge\frac{1}{x+1}+\frac{3}{y+3}\ge2\sqrt{\frac{3}{\left(x+1\right)\left(y+3\right)}}\)(1)

từ giả thiết :\(\frac{1}{x+1}\le\frac{z}{z+2}-\frac{3}{y+3}\Leftrightarrow1-\frac{1}{x+1}\ge1-\frac{z}{z+2}+\frac{3}{y+3}\)

\(\Leftrightarrow\frac{x}{x+1}\ge\frac{2}{z+2}+\frac{3}{y+3}\)

Áp dụng bất đẳng thức cauchy 1 lần nữa: \(\frac{x}{x+1}\ge\frac{2}{z+2}+\frac{3}{y+3}\ge2\sqrt{\frac{6}{\left(z+2\right)\left(y+3\right)}}\)(2)

tương tự ta cũng có: \(\frac{y}{y+3}\ge2\sqrt{\frac{2}{\left(z+2\right)\left(x+1\right)}}\)(3),

cả 2 vế các bất đẳng thức (1),(2)và (3) đều dương, nhân vế với vế: 

\(\frac{xyz}{\left(x+1\right)\left(y+3\right)\left(z+2\right)}\ge\frac{8.6}{\left(x+1\right)\left(z+2\right)\left(y+3\right)}\)

\(\Leftrightarrow xyz\ge48\)

Dấu = xảy ra khi x=2,y=6,z=4 hay a=1,b=5,z=3

Bình luận (0)